The two triangles are similar.

What is the value of x?



Enter your answer in the box.

x =

Answers

Answer 1

Answer:

x=4

Step-by-step explanation:

[tex]\frac{16}{12}= \frac{6x}{5x-2}[/tex]

assuming that the triangles are similar, we can create this ratio of 16 or 12+4 to 6x and 12 to 5x-2

we can cross multiply to get 16(5x-2)=6x(12)

that leaves you with 80x-32=72x

we can then subtract 80x to both sides to get -32= -8x

solve by diving -8 from both sides which means that x=4

Answer 2

Answer:

[tex]this \: triangles \: are \: similar \: so \\ sides \: must \: be \: in \: a \: ratio \\ \frac{16}{12} = \frac{6x}{5x - 2} \\ \frac{4}{3} = \frac{6x}{5x - 2} \\ 4(5x - 2) = 3 \times 6x \\ 20x - 8 = 18x \\ x \: terms \: togather \\ 20x - 18x = 8 \\ 2x = 8 \\ x = \frac{8}{2} \\ x = 4 \\ thank \: you[/tex]


Related Questions

1->dương vô cùng 1/x*(9+lnx^2)dx

Answers

It looks like you are trying to compute the improper integral,

[tex]I = \displaystyle\int_1^\infty \dfrac{\mathrm dx}{x(9+\ln^2(x))}[/tex]

or some flavor of this. If this interpretation is correct, substitute u = ln(x) and du = dx/x. Then

[tex]I = \displaystyle\int_0^\infty \dfrac{\mathrm du}{9+u^2} \\\\ = \frac13\arctan\left(\frac u3\right)\bigg|_{u=0}^{u\to\infty} \\\\ = \frac13\lim_{u\to\infty}\arctan\left(\frac u3\right) \\\\ = \frac13\times\frac\pi2 = \boxed{\frac\pi6}[/tex]

I will give brainliest if you answer properly.

Answers

Answer:

See below

Step-by-step explanation:

a)

[tex]2\sin(x) +\sqrt{3} =0 \implies 2\sin(x)=-\sqrt{3} \implies \boxed{\sin(x)=-\dfrac{\sqrt{3}}{2} }[/tex]

[tex]\therefore x=\dfrac{4\pi }{3}[/tex]

But note, as sine does represent the [tex]y[/tex] value, [tex]\dfrac{5\pi }{3}[/tex] is also solution

Therefore,

[tex]x=\dfrac{4\pi }{3} \text{ and } x=\dfrac{5\pi }{3}[/tex]

This is the solution for [tex]x\in[0, 2\pi ][/tex], recall the unit circle.

Note: [tex]\sin(x)=-\dfrac{\sqrt{3}}{2} \implies \sin(x)=\sin \left(\pi +\dfrac{\pi }{3} \right)[/tex]

b)

[tex]\sqrt{3} \tan(x) + 1 =0 \implies \tan(x) = -\dfrac{1}{\sqrt{3} } \implies \boxed{ \tan(x) = -\dfrac{\sqrt{3} }{3} }[/tex]

Once

[tex]\tan(x) = -\dfrac{\sqrt{3} }{3} \implies \sin(x) = -\dfrac{1}{2} \text{ and } \cos(x) = \dfrac{\sqrt{3} }{2}[/tex]

As [tex]\tan(x) = \dfrac{\sin(x)}{\cos(x)}[/tex]

[tex]\therefore x=-\dfrac{\pi }{6}[/tex]

c)

[tex]4\sin^2(x) - 1 = 0 \implies \sin^2(x) = \dfrac{1}{4} \implies \boxed{\sin(x) = \pm \dfrac{\sqrt{1} }{\sqrt{4} } = \pm \dfrac{1}{2}}[/tex]

Therefore,

[tex]\sin(x)=\dfrac{1}{2} \implies x=\dfrac{\pi }{6} \text{ and } x=\dfrac{5\pi }{6}[/tex]

[tex]\sin(x)=-\dfrac{1}{2} \implies x=\dfrac{7\pi }{6} \text{ and } x=\dfrac{11\pi }{6}[/tex]

The solutions are

[tex]x=\dfrac{\pi }{6} \text{ and } x=\dfrac{5\pi }{6} \text{ and }x=\dfrac{7\pi }{6} \text{ and } x=\dfrac{11\pi }{6}[/tex]

Question 3 of 10
What is the value of p?
V140
140°
90-
A. 50°
ООО
B. 90°
C. 60°
D. 40°

Answers

Answer:

A. 50º

Step-by-step explanation:

we are given the exterior angles 140º and 90º

exterior angles + corresponding interior angles = 180º

that means the two other angles of the triangle are:

180 - 140 = 40º

and

180 - 90 = 90º

the sum of interior angles in a triangle = 180

p = 180 - (40 + 90)

p = 180 - 130

p = 50º

The average mileage per gallon for cars built since 1940 approximates to the following curve 0.0075*t^2-.2672*t+14.8 where t is year -1940.

Answer the following questions:

What is the expected MPG in 2025?
How about 2050?
Is this a valid function?
Is there a top end to MPG?

Answers

9514 1404 393

Answer:

46.3 in 202576.2 in 2050

Step-by-step explanation:

The attached shows the predicted mileage for cars built in 2025 to be 46.3 mpg, 76.2 mpg for cars built in 2050.

__

No doubt, the function is valid over the time period used to derive it. It may or may not be valid for predicting MPG beyond that period.

Virtually any function that predicts future increases without bound will turn out to be unreliable at some point. In this universe, there are always limits to growth.

Samantha bought m candies at the store. There are n candies in a pound, and each pound costs c dollars. Write an expression for how much Samantha paid.

Answers

Answer:

total = m/n * c

m/n gives u the number of pounds u have bought, multplied by the cost of the candies per pound gives u the total amount of money she paid

5 less than three times a number is 37 what is the number​

Answers

Answer:

x = 14

General Formulas and Concepts:

Pre-Algebra

Equality Properties

Multiplication Property of Equality Division Property of Equality Addition Property of Equality Subtraction Property of Equality

Step-by-step explanation:

Step 1: Define

Identify

3x - 5 = 37

Step 2: Solve for x

[Addition Property of Equality] Add 5 on both sides:                                    3x = 42[Division Property of Equality] Divide 3 on both sides:                                 x = 14

Solve triangles: angle bisector theorem
DAC = BAD.
What is the length of CD?
Round to one decimal place.

Answers

Answer:

Step-by-step explanation:

CD/6.5 = 2.6/4.9    This is the result of the angle bisector theorem.

The theorem basically says that the side opposite the angle being bisected is divided  the ratio of the sides enclosing the angle.

Multiply both sides of the proportion by 6.5

CD = 2.6 * 6.5 / 4.9

CD = 3.4489

CD = 3.4 rounded.

If the lengths of the legs of a right triangle are 5 and 12, what is the length of the hypotenuse?

Answers

Answer:

13

Step-by-step explanation:

If we have a right triangle, we can use the Pythagorean theorem to find the hypotenuse

a^2+b^2 = c^2 where a and b are the legs and c is the hypotenuse

5^2 + 12^2 = c^2

25+144= c^2

169 = c^2

Take the square root of each side

sqrt(169) = sqrt(c^2)

13= c

Answer:

The length of the hypotenuse is 13.

Step-by-step explanation:

[tex]a^{2}[/tex] = [tex]b^2 + c^2[/tex]

[tex]a^2 = 12^2 + 5^2[/tex]

[tex]a^2 = 144 + 25[/tex]

[tex]a^2 = 169[/tex]

a=[tex]\sqrt{169}[/tex]

a= 13

Here we use the idea of the Pythagoras' theorem. Which suggests that [tex]a^{2}[/tex] = [tex]b^2 + c^2[/tex] in which  [tex]a^{2}[/tex] is the hypotenuse of the triangle and [tex]b^2[/tex] and [tex]c^{2}[/tex] are the two other lengths of the triangle.

HOPE THIS HELPED

Drag each factor to the correct location on the image.
If p(1) = 3, p(-4) = 8, p(5) = 0, p(7) = 9, p(-10) = 1, and p(-12) = 0,
P(x).

Answers

Answer:

(x-7) and (x+12) are the factors and the rest are non factors...

3.06 as. a fraction PLEASE HELP​

Answers

Answer:

153/50

Step-by-step explanation:

3.06

Rewriting as

There are two numbers after the decimal so we put the number over 100

306/100

Divide top and bottom by 2

153/50

To write 3.06 as a fraction you have to write 3.06 as numerator and put 1 as the denominator. Now you multiply numerator and denominator by 10 as long as you get in numerator the whole number.

3.06 = 3.06/1 = 30.6/10 = 306/100

And finally we have:

3.06 as a fraction equals 306/100

I need help Plz help

Answers

2 Eyes =2*(1*1/2)/2 = 0.5ft^2
Nose = ((1+1/2)*1)/2 = 0.75^2
Mouth = 3*(1ft/2)/2= 0.75ft^2
Board = 2/2/3*3 =8ft^2
Area = 8-0.5-0.75-0.75 = 6ft^2

A ball is thrown from an initial height of
1 meter with an initial upward velocity of
1 m/s. The ball's height h
(in meters) after t
seconds is given by the following. h=1+30t-5t^2
Find all values of t
for which the ball's height is 11
meters.

Round your answer(s) to the nearest hundredth.

Answers

Answer:

Step-by-step explanation:

If we are looking for the times that the ball was 11 meters off the ground, we sub in 11 for the height on the left and solve for t:

[tex]11=-5t^2+30t+1[/tex] and

[tex]0=-5t^2+30t-10[/tex] and factor this however it is you are factoring in class to solve for t to get

t = .35 seconds and t = 5.6 seconds

Because the ball reaches this point in its way up and then passes it again on its way down, the ball will have 2 times at this height.

Naomi invested $3,425 in an account that
pays 3% simple interest. what was the total
balance of the account after 15 years?

Answers

Answer:

$4,966.25

Step-by-step explanation:

3 x 15 = 45

After 15 years, Naomi would have earned a total of 45% interest rate.

3,425 x 1.45 = 4,966.25

Don't use .45 as the multiplier

3,425 x .45 = 1,541.25 <- incorrect

Answer: 4,966.25

Explanation: start by diving 3425 by .03 to get the amount of interest it is yearly giving you 102.75 and then multiply that by 15 since that’s the amount of years and get 1541.25 then add 3425 and 1541.25 together to get the final product of 4,966.25

Part E
1e. Subtract the binomial 12y2 – 4y3 from the trinomial 7y - 2y3 + 5y2

Answers

Answer:

2y^3-7y^2+7y

Step-by-step explanation:

7y - 2y^3 + 5y^2 - (  12y^2 – 4y^3)

Distribute the minus sign

7y - 2y^3 + 5y^2 - 12y^2 + 4y^3

Combine like terms

2y^3-7y^2+7y

How is the series 6+13+20+...+111 represented in summation notation?

Answers

Notice that

6 + 7 = 13

13 + 7 = 20

so if the pattern continues, the underlying sequence in this series is arithmetic with first term a = 6 and difference d = 7. This means the k-th term in the sequence is

a + (k - 1) d = 6 + 7 (k - 1) = 7k - 1

The last term in the series is 111, which means the series consists of 16 terms, since

7k - 1 = 111   ==>   7k = 112   ==>   k = 16

Then in summation notation, we have

[tex]\displaystyle 6+13+20+\cdots+111 = \boxed{\sum_{k=1}^{16}(7k-1)}[/tex]

there is a 400 meter track. tom rides a bike at the speeed of 450 meters/minute. mike runs at the speed of 250 meters/minute. if both of them set out at the same time and same place, how soon will they meet for the first time?

Answers

Answer: Distance: 2250

Step-by-step explanation:

Find GCF of 450 and 250

PLS HELP QUICK IM BEGGINGGGG!!!!! PLEASE HELP ME!!

The following box plot represents the heights of the students in Mr. Taylor's fourth grade math class.

In a complete sentence, answer the following question:
One of the values in this data set is 138. In this box plot, what does this value mean?

Answers

Answer:

The value 138 means that this height (138cm) is less than the average height of a 4th grader.

Answer: No credit wanted

Step-by-step explanation:

The other guy is completely right.

The length of the shadow of a flagpole was found to be 72 feet. The shadow of a 3 foot picket fence in line with the flagpole was 4 feet. What is the height of the flagpole.

Answers

Answer:

54ft

Step-by-step explanation:

if 3foot is 4ft tall

then a shadow of 72 ft will be how tall?= x

4x =3ft by 72ft

4x= 216ft

x= 216/4

×=54

A toddler is allowed to dress himself on Mondays, Wednesdays, and Fridays. For each of his shirt, pants, and shoes, he is equally likely to put it on correctly as incorrectly. Getting these articles of clothing on correctly are independent of each other. On the other days, the mother dresses the toddler with 100% accuracy. Given that the toddler is correctly dressed, what is the probability that today is Monday

Answers

Answer:

0.0286 = 2.86% probability that today is Monday.

Step-by-step explanation:

Conditional Probability

We use the conditional probability formula to solve this question. It is

[tex]P(B|A) = \frac{P(A \cap B)}{P(A)}[/tex]

In which

P(B|A) is the probability of event B happening, given that A happened.

[tex]P(A \cap B)[/tex] is the probability of both A and B happening.

P(A) is the probability of A happening.

In this question:

Event A: Dressed correctly

Event B: Monday

Probability of being dressed correctly:

100% = 1 out of 4/7(mom dresses).

(0.5)^3 = 0.125 out of 3/7(toddler dresses himself). So

[tex]P(A) = 0.125\frac{3}{7} + \frac{4}{7} = \frac{0.125*3 + 4}{7} = \frac{4.375}{7} = 0.625[/tex]

Probability of being dressed correctly and being Monday:

The toddler dresses himself on Monday, so (0.5)^3 = 0.125 probability of him being dressed correctly, 1/7 probability of being Monday, so:

[tex]P(A \cap B) = 0.125\frac{1}{7} = 0.0179[/tex]

What is the probability that today is Monday?

[tex]P(B|A) = \frac{P(A \cap B)}{P(A)} = \frac{0.0179}{0.625} = 0.0286[/tex]

0.0286 = 2.86% probability that today is Monday.

What is the value of x?

Answers

Answer:

x=30°

hopefully this answer can help you to answer the next question

Instructions: State what additional information is required in order
to know that the triangles in the image below are congruent for the
reason given
Reasory. AAS Postulate

Answers

Answer:

YWX = DFE

Step-by-step explanation:

AAS means angle angle side. so, we need 2 angles and 1 side.

we have 1 side and one angle confirmed.

so, we need one of the other two angles (W or Y vs. F or D) confirmed.

they probably want W and F as answer, as Y and D would make it a special case of AAS : ASA.

Add :-

a+2b-3c, -3a+b+2cand 2a -3b+c​

Answers

Answer:

[tex]a + 2b -3 c + - 3a + b + 2c + 2a - 3b + c \\ = a - 3a + 2a + 2b + b - 3b - 3c + 2c + c \\ 0a + 0b + 0c \\ thank \: you[/tex]

a+2b-3c+(-3a+b+2c) +(2a-3b+c)

=a+2b-3c-3a+b+2c+2a-3b+c

=a-3a+2b+2b+b-3b-3c++2b+c

=0a+0b+0c

=0

Therefore, the addition of the expressions, a+2b-3c+(-3a+b+2c) +(2a-3b+c) is zero or 0.

To know more about algebraic expressions

https://brainly.com/question/28307605

Use the tables to answer the question.
How can you determine which store offers a better price on guitar lessons?
A. You cannot determine this from the information given.
B. Compare the most expensive price from each store’s table.
C. Compare the least expensive price from each store’s table.
D. Find the cost of 1 week of guitar lessons for each store and compare.

Answers

Answer:

D. Find the cost of 1 week of guitar lessons for each store and compare.

Step-by-step explanation:

Find the cost for 1 week of guitar lessons by finding the slope of the data.

[tex]\frac{y_2-y_1}{x_2-x_1}\\\\ A.)\frac{132-66}{6-3}=\frac{66}{3}=22\\\\ B.)\frac{168-84}{8-4}=\frac{84}{4}=21[/tex]

Therefore, Store B would be the best choice.

4. Manuel swims at a speed of 1 yard per second. How many feet per minute does he swim?​

Answers

Answer:

180 Feet Per Minute

Step-by-step explanation:

Please mark me as brainliest and rate 5 stars!

180 feet per minute.

Evaluate:
[tex]{ \int \limits^\pi_{ \frac{1}{4}\pi}{ {e {}^{2 \sigma} (\sqrt{1 - { \sigma}^{2} } ) d \sigma}}}[/tex]

Answers

Answer:

hope this answer helps.

QUESTION 3.1 POINT
An investment pays 25% interest compounded monthly. What percent, as a decimal, is the effective annual yied? Enter your
answer as a decimal rounded to four decimal places.

Answers

9514 1404 393

Answer:

  0.2807

Step-by-step explanation:

The relationship between the effective annual yield (e) and the nominal annual interest rate (r) compounded n times per year is ...

  e = (1 +r/n)^n -1

  e = (1 +0.25/12)^12 -1 = 0.2807 . . . . . . about 28.07%

The resistors produced by a manufacturer are required to have an average resistance of 0.150 ohms. Statistical analysis of the output suggests that the resistances can be approximated by a normal distribution with known standard deviation of 0.005 ohms. We are interested in testing the hypothesis that the resistors conform to the specifications.

Requied:
a. Determine whether a random sample of 10 resistors yielding a sample mean of 0.152 ohms indicates that the resistors are conforming. Use alpha = 0.05.
b. Calculate a 95% confidence interval for the average resistance. How does this interval relate to your solution of part (a)?

Answers

Answer:

a) The p-value of the test is 0.2076 > 0.05, which means that the sample indicates that the resistors are conforming.

b) The 95% confidence interval for the average resistance is (0.147, 0.153). 0.152 is part of the confidence interval, which means that as the test statistic in item a), it indicates that the resistors are conforming.

Step-by-step explanation:

Question a:

The resistors produced by a manufacturer are required to have an average resistance of 0.150 ohms.

At the null hypothesis, we test if this is the average resistance, that is:

[tex]H_0: \mu = 0.15[/tex]

We are interested in testing the hypothesis that the resistors conform to the specifications.

At the alternative hypothesis, we test if it is not conforming, that is, the mean is different of 0.15, so:

[tex]H_1: \mu \neq 0.15[/tex]

The test statistic is:

[tex]z = \frac{X - \mu}{\frac{\sigma}{\sqrt{n}}}[/tex]

In which X is the sample mean, [tex]\mu[/tex] is the value tested at the null hypothesis, [tex]\sigma[/tex] is the standard deviation and n is the size of the sample.

0.15 is tested at the null hypothesis:

This means that [tex]\mu = 0.15[/tex]

Sample mean of 0.152, sample of 10, population standard deviation of 0.005.

This means that [tex]X = 0.152, n = 10, \sigma = 0.005[/tex]

Value of the test statistic:

[tex]z = \frac{X - \mu}{\frac{\sigma}{\sqrt{n}}}[/tex]

[tex]z = \frac{0.152 - 0.15}{\frac{0.005}{\sqrt{10}}}[/tex]

[tex]z = 1.26[/tex]

P-value of the test and decision:

The p-value of the test is the probability of the sample mean differing from 0.15 by at least 0.152 - 0.15 = 0.002, which is P(|z| > 1.26), given by two multiplied by the p-value of z = -1.26.

Looking at the z-table, z = -1.26 has a p-value of 0.1038.

2*0.1038 = 0.2076

The p-value of the test is 0.2076 > 0.05, which means that the sample indicates that the resistors are conforming.

Question b:

We have to find our [tex]\alpha[/tex] level, that is the subtraction of 1 by the confidence interval divided by 2. So:

[tex]\alpha = \frac{1 - 0.95}{2} = 0.025[/tex]

Now, we have to find z in the Z-table as such z has a p-value of [tex]1 - \alpha[/tex].

That is z with a p-value of [tex]1 - 0.025 = 0.975[/tex], so Z = 1.96.

Now, find the margin of error M as such

[tex]M = z\frac{\sigma}{\sqrt{n}}[/tex]

In which [tex]\sigma[/tex] is the standard deviation of the population and n is the size of the sample.

[tex]M = 1.96\frac{0.005}{\sqrt{10}} = 0.003[/tex]

The lower end of the interval is the sample mean subtracted by M. So it is 0.15 - 0.003 = 0.147.

The upper end of the interval is the sample mean added to M. So it is 0.15 + 0.003 = 0.153.

The 95% confidence interval for the average resistance is (0.147, 0.153). 0.152 is part of the confidence interval, which means that as the test statistic in item a), it indicates that the resistors are conforming.

There are two machines available for cutting corks intended for use in wine bottles. The first produces corks with diameters that are normally distributed with mean 3 cm and standard deviation 0.10 cm. The second machine produces corks with diameters that have a normal distribution with mean 3.04 cm and standard deviation 0.04 cm. Acceptable corks have diameters between 2.9 cm and 3.1 cm. What is the probability that the first machine produces an acceptable cork

Answers

Answer:

0.6827

Step-by-step explanation:

Given that :

Mean, μ = 3

Standard deviation, σ = 0.1

To produce an acceptable cork. :

P(2.9 < X < 3.1)

Recall :

Z = (x - μ) / σ

P(2.9 < X < 3.1) = P[((2.9 - 3) / 0.1) < Z < ((3.1 - 3) / 0.1)]

P(2.9 < X < 3.1) = P(-1 < Z < 1)

Using a normal distribution calculator, we obtain the probability to the right of the distribution :

P(2.9 < X < 3.1) = P(1 < Z < - 1) = 0.8413 - 0.1587 = 0.6827

Hence, the probability that the first machine produces an acceptable cork is 0.6827

-5 + 3 and also what is 1/4 of 24
What is the answer i am struggling

Answers

Answer:

-5+3=-2

1/4 of 24 = 6

Step-by-step explanation:

PLEASEE HELP ME ASAPPP (geometry)

Answers

Answer:AE=EC và BF=FC => EF là đường trung bình của tam giác ABC

=> EF // và bằng 1/2 AB

=> AB = 16

Step-by-step explanation:

Answer:

AB=16

Step-by-step explanation:

Midsegment Theorem states that the segment connecting the midpoints of two sides of a triangle is parallel to the third side and half as long.

The mid-segment of a triangle, which joins the midpoints of two sides of a triangle, is parallel to the third side of the triangle and half the length of that third side of the triangle.

AD=DB

AD+DB=AB=2EF

AB=2×8=16

Other Questions
Convert 7 6/7 into an improper fraction. At which times could Rory's phone have been plugged into the charger? Select three options. A hole the size of a photograph is cut from a red piece of paper to use in a picture frame. On a coordinate plane, 2 squares are shown. The photograph has points (negative 3, negative 2), (negative 2, 2), (2, 1), and (1, negative 3). The red paper has points (negative 4, 4), (4, 4), (4, negative 4), and (negative 4, negative 4). What is the area of the piece of red paper after the hole for the photograph has been cut? 17 square units 25 square units 39 square units 47 square units Mark this and return Answer pls. Can someone help me with this please !! Find the area of the figure. A chocolate factory has three classifications for the candies that it makes:Caramels (12 varieties). please help!!!Milk Chocolate (6 varieties) Dark Chocolate (9 varieties)1. Set up the NOTATION to figure out the different ways to choose 7 of the 12 Caramels2. Set up the NOTATION to figure out the different ways to choose 3 of the 6 Milk Chocolates3. Set up the NOTATION to figure out the different ways to choose 5 of the 9 Dark Chocolates4. A customer has ordered an assortment to consist of seven types of caramels, three types of chocolate, and five types of dark chocolates.How many such assortments are possible? Factor: x^3-2x^2-8x1. (x-4)(x+2)2. x(x+4)(x-2)3. x(x-4)(x+2)4. (x-4)(x^2+2x) 1) A book contains 192 pages. A boy reads x complete pages everyday.if he has not finished the book after 10days, find the highest possible value of x What is the volume of this rectangular pyramid?______ cubic feet People from what continent want to rob mansa musa and distroy his riches the most? The stratum spinosum is a deep epidermal layer that ________keratin.o produceso destroys o recycles(there is a pic of the answer I got wrong :) ) Show all work to solve the equation for x. If a solution is extraneous, be sure to identify it in your final answer.square root of the quantity x plus 7 end quantity minus 1 equals x Question 10 The hypotenuse of a right triangle is I m longer than the longer leg. The other leg is 7 m shorter than the longer leg. Determine the lengths of the three sides of the triangle. (3 marks) What aspects of semantics did you enjoy most? Discuss it? PLS HELP ME ON THIS QUESTION I WILL MARK YOU AS BRAINLIEST IF YOU K NOW THE ANSWER!!The entire group of objects or individuals under consideration in a survey is the ________________.A. populationB. establishmentC. corporationD. entity What is the intersecting point of these two lines?1. y = 3/4x + 62. y = x + 1 Henry says, "it doesn't seem delicate, some how,". it refers(A) The bureau of grandfather.( B ) Shabby old chest of drawers. (C) To bring down the bureau from grandfather's room2. What did Slater's want to bring downplz ans fastly What kind of evidence exists for the Bantu expansion?A. Agricultural evidenceOB. Religious evidenceC. Written evidenceO D. Linguistic evidence Ba ng phn gic ca tam gic ABC gp nhau ti O pht biu no sau y ngA Ao lun vung gc vi BCB OA=OB=OCC Ao lun i qua trung im BCD O cch u ba cnh ca tam gic